LSAT and Law School Admissions Forum

Get expert LSAT preparation and law school admissions advice from PowerScore Test Preparation.

 cmorris32
  • Posts: 92
  • Joined: May 05, 2020
|
#75318
Hello!

I have a question about Question 12.2 on page 111. I don't really understand numerical distribution in this case, and I don't understand why the distribution is 3-1-1-1-1-1-1. I read the explanation on page 131 but I still don't understand. Can you please elaborate on why the correct answer is C?

Thank you!
User avatar
 Dave Killoran
PowerScore Staff
  • PowerScore Staff
  • Posts: 5853
  • Joined: Mar 25, 2011
|
#75320
Hi C,

I'll do what I can, but the explanation I made on page 128-129 is, to me, the most straightforward way to do it and I'm not sure how much I can improve on that but I'll try :-D The distribution here is one about the number of patients scheduled for each day. So, let's start with what we know:

  • Number of patients = 9
    How many days are they being scheduled for? 7
    How many must be scheduled each day? At least 1
So, if you think about this as if you were in charge of scheduling. You've got 9 patients to assign over 7 days. How do you do it?

Well, the first thing is that you need 1 patient per day to satisfy the stated requirement. this would look like:

  • Monday = 1
    Tuesday = 1
    Wednesday = 1
    Thursday = 1
    Friday = 1
    Saturday = 1
    Sunday = 1
That by itself assigns 7 of the 9 patients. So, what can we do with those other two patients? We can assign them anywhere, and we can assign them both to one day or we can split them up and assign them to different days. For simplicity sake, let's say we assigned both of the extra patients to one day, Friday:

  • Monday = 1
    Tuesday = 1
    Wednesday = 1
    Thursday = 1
    Friday = 3
    Saturday = 1
    Sunday = 1
So, the question here is, can we ever get more than 3 patients on Friday (or any other day)? No, we've now assigned all 9 patients and there are no more left to assign. So, on Friday (or any day), the maximum number is 3. Thus, (C) is the correct answer in that problem.

Note that we could have assigned that 3 anywhere, to any day, but that every other day must then be 1 patient. That's where we get the unfixed 3-1-1-1-1-1-1 distribution from.

If the above doesn't make sense, try making up a schedule on your own. Create 9 patients and then assign them to different days using the "at least one" rule above. There are only two ways to do it!

Please let me know if that helps. Thanks!
 cmorris32
  • Posts: 92
  • Joined: May 05, 2020
|
#75398
Hi Dave!

Thank you so much for this explanation. Using the example of Friday helps! :-D :-D
 leslie7
  • Posts: 73
  • Joined: Oct 06, 2020
|
#80618
"Note that we could have assigned that 3 anywhere, to any day, but that every other day must then be 1 patient. That's where we get the unfixed 3-1-1-1-1-1-1 distribution from."

Hello, I was caught off by this because I understood the distribution numbers at the back of my mind but when I was looking at the answer choices I was most fixated on the days the answers were presenting. So I couldn't pick any of the answers cuz they were not "must be true" - its not "must be true" that on friday there was a max of 3 , like you said it could have been any day. If I had this question on a real LSAT i'm afraid I wouldn't understand how to select the AC even though I technically knew the answer... how do you avoid a confusion like this when selecting the correct answer? e.g how to avoid becoming fixated on the fact that the answer choice focused on friday when it could be any day - is that really a must be true question?

Clarity on this would be greatly appreciated. I'm sure its stemming from my mis understanding of what a MBT is but maybe if someone explains it it'll shed light on what I'm mis interpreting .

Ty so much .
 Jeremy Press
PowerScore Staff
  • PowerScore Staff
  • Posts: 1000
  • Joined: Jun 12, 2017
|
#80655
Hi leslie,

Think of it this way: that 3 is a general maximum in the game, true, but it's a maximum that applies to every specific day as well. 3 is the most you could have on any given day of the week. So doesn't that mean that, since the general maximum applies to any specific day, that maximum must be true of those specific days as well? So, it must be true that the maximum number to go on Sunday is 3. And the maximum for Monday also must be 3. And the maximum for Tuesday also must be 3, etc.

Just because a Must Be True answer choice refers to a specific instance of a more general rule, that doesn't make it wrong. It still has to be true, so that answer choice C is the correct answer.

I hope this helps!
 leslie7
  • Posts: 73
  • Joined: Oct 06, 2020
|
#80745
Hi Jeremy,

This was helpful - I do see what you are saying, I appreciate your response ty! :)

Get the most out of your LSAT Prep Plus subscription.

Analyze and track your performance with our Testing and Analytics Package.